Answer (C) is correct . Marginal revenue exceeds marginal cost for the fourth but not the fifth unit. Production should continue until marginal cost equals marginal revenue. Accordingly, four units should be produced.
Answer (A) is incorrect because Two units of production is not the highest level of production where marginal revenue exceeds marginal costs. Production should continue up to and until marginal revenue equals marginal cost. Answer (B) is incorrect because Four units could be produced and still increase the company’s profits. Answer (D) is incorrect because At 5 units of production, the marginal cost ($107) exceeds the marginal revenue ($90) so the company would lose income by producing this unit.
|